EMI Advanced SRG

You might also like

Download as pdf
Download as pdf
You are on page 1of 86
ELECTROMAGNETIC INucTion E= -N AG Forocays hans of EME > whan magwitic paw possing Ube ough & Heep chon with bine or magnalicc dima of fone on hut a concluching wih, am im 48 prodiacech im Yu trope On im Thal wine Les tm] in Cal inducsol wd Y te Linuit ja Lord, dun Un tnt will be Coli in dul Urumt - = ‘he mmagnidtucl, of imdiscad ome] 4 myual fo Aa rate. of change of fam wrt tne in ‘p fa N tia 3 es — NAD) e- -dg dt ot “ sigm indicat that wmf wilt be inadhreaol mw suck a von Ba UE iLL ppb te Change of fin @ _byvarying the magnitude of jj with time (illustrated in Figure) | 36 8 ren Figure : Inducing emfby varying the magnetic field strength (i) byvarying the magnitude of { , ic, the area enclosed by the loop with time (illustrated in Figure) Anan A Figure : Inducing emfby changing the area of the loop Gii) varying the angle between j and thearea vector with time (illustrated in Figure) Figure : Inducing emf by varying the angle between Band. A Lenz's Law: ‘The direction of the induced current is determined by Lenz’s law: The induced current produces magnetic fields which tend to oppose the change in magnetic flus that induces such currents, ‘To illustrate how Lenz’s law works, le’s consider a conducting loop placed in a magnetic field. We follow the procedure below: Define positive direction for the area vector & « Assuming that jis uniform, take the dot product of jj and . Thisallows forthe determination of the sign ofthe magnetic flux ¢, Obtain therateofflux change deb, / dt by differentiation. There are three possibilities: dy at /< = induced emf ¢>0 [> 0=> induced emf <0 = induced emt Determine the direction ofthe induced curren usingthe right-hand rule. With your thumb pointing inthe direction of & ,curl the fingers around the closed loop. The induced current flows inthe same direction asthe way your fingers curl ie > 0, and the opposite direction if <0, as shown in Figure Figure : Determination ofthe direction of induced current by the right-hand rule V2 vo bed a BT He time fej= AT. Aga “ied at dt 2 A de at Midge Qo ABL = Int}, at «& Ri SN — Jel = 098 = axto iw ak t= 2° uA = 2oV es NI 20 ace lb y 2 loT/s ot ob BSL e- DAB = ax 'O=ROV 30V “ we 1 F i [+ > boca a cee i a 2 t 7 rea Ki dp BdA = wei tdx 21TH atl ge wih Pde = Heil [i io-Ma] = |} F Dar 257 ze 2 uh ft ae a ae] ele = Maid [iy a at = Wait fa —DIy “an | alate) 29 ~ ak bo Qnalat) x £ = Blu snd blo E Raik Keel Ex printed a > E-Ble fat is bly R co pee od it to ra with Comsat velocity, Ain 74 at = f = ihe fas dined by fob Fe th P- thB ur “60 P- ody? Frm cassspation im raistomen = (= TR | = OLE R f- Batu K R Lt of tu too 9 we find vo ag? of ©: AL time € > a A = Me (1B =-m au at t duis - BL" ot mR > fin oJ = BLL mR b(2) « hate uy Canal -Kt = ee vr UC ‘ TAR rte T21 = 28 ve ue kK Gh AL t=0 DB UO v » pws : 5 Cdegd rod neve To nat uw t tlt - Ke Us Uwe = Ue t S -kt ds ~ ue 9 Jes - fue ue dé ° ° Asking Questions : @ o) @ ) Foop for Taou GTS. ‘A bar magnet falls through a circular loop, as shown in Figure ' ; | co Describe qualitatively the change in magnetic lux through the loop when the bar magnetis above and below the loop. Make a qualitative sketch of the graph of the induced current inthe loop as a funtion oftime, choosing, Tobe positive when its direction is counterclockwise as viewed from above. ‘Two circular loops 4 and B have their planes parallel to each other, as shown in Figure. 0 Loop 4 has current moving in the counterclockwise direction, viewed from above, Ifthe current in loop 4 decreases wit time, what isthe direction of the induced current in loop B? Will the two loops attractor repel each other? Ifthe current in loop increases with time, whats the direction of the induced current in loop B? Will the ‘two loops attractor repel each other? ‘A spherical conducting shells placed ina time-varying magnetic field, Is there an induced current along the equator? Arectangular loop moves across a uniform magnetic field but the induced current is zero, How is this possible? &, ‘wire bent as a parabola y= ax" is located in a uniform magnetic field of induction B, the vector B being ‘perpendicular to the plane x, y. Atthe moment =0a connector stars sliding translationwise from the parabola apex with a constant acceleration « (Fig,). Find the emf of electromagnetic induction in the loop thus formed asa function of y. vO Vw 4 cal J é—%X%— 32 | __, &, YA rectangular loop witha sliding connector of length is located ina uniform magnetic field perpendicular ‘to the loop plane (Fig.). The magnetic induction is equal to B. The connector has an electric resistance R, the sides AB and CD have resistances R, and R, respectively. Neglecting the self-induetance of the Joop, find the current flowing in the connector during its motion with a constant velocity v. a D 8 ine wire with a resistance of r per unit length is bent to form an angle 20. A rod of the same wire perpendicular to the angle bisector (of 2) forms a closed triangular loop. This loop is placed in a uniform magnetic field of induction B. Calculate the current in the wires when the rod moves at a constant speed V. ‘ZLoop A of radius r(r << R) moves towards loop B with a constant velocity V in such a way that their planes are always parallel. What is the distance between the two loops (x) when the induced emf in oop Ais maximum F(r << R) Prout a ae Aga BA awe Pad 1 VA ga TOR mA aa eR GAS aa we TAA aed 81 dT eB Fer GO (x) TATA eM ITT af Go_AR ARG Rozora sitar a we A copper connector of mass m slides dawn two smooth copper bars, setat an angle o: to the horizontal, due to gravity (Fig,). At the top the bars are interconnected through a resistance R. The separation, L Thesystemis located in uniform magnetic field ofinductionB, perpendicular to the plane in which the connector slides. The resistances of the bars, the connector and the sliding contacts, as-welhastheselfinduetance.ofthe loop,-areassamredteberegtigible, Find the steady-state velocity of the connector. usd 4270 Ve —_ of mass m and length ¢ can slide freely on a pair of fixed, smooth, vertical rails (figure). Amagnetic field Beexists in the region in the direction perpendicular to the plane of the rails. The rails are connected at the top. end by an initially uncharged capacitor of capacitance C. Find the velocity of the wire at any time (t) after released. Neglecting any electric resistance. (initial velocity of wire is zero) Prarh, Seales osaq veld & get oe earaT maen sag Ca aT Ba wa S Powe aT B (Pea) | veka & cao aaa we easy aa Bafta 4 2) veftal wr a nit Cara data Gres F reaper) GT 8 1 Ara aR a Te ATT KE TSA a rea Foe THT ae ATR aT AT BT A | (are a aren 41 ea 8) y . ww Two parallel long smooth conducting rails separated by a distance ‘= 10 cm are connected by a movable conducting connector of mass m = 4mg. Terminals of the rails are connected by the resistor R= 20 & the capacitor C = 1F as shown. Auniform magnetic field B = 20T perpendicular to the plane of the rails is switched on. The connector is dragged by a constant force F=10N. The speed of the connector as function of time if the force F is applied at t= 0 is equal to y ~ 5(1—e 10") m/s Find the value of x. we Ww Qin the fguié COEF sa fixed conducting smooth frame in vertical plane. A conducting uniform rod GH of ‘4mass'm’ = 1 gcan move vertically and smoothly without losing contact with the frame, GH always remains horizontal. Itis given velocity 'u= 1 m/s’ upwards and released. Taking the acceleration due to gravity as'g° and assuming that no resistance is present other than 'R’. Time taken by rod to reach the highest point is. ent equal to “a8 second Find out value of x. Pea 8 CDEF vo ur ete organ, area Sha Gh ed cet A cea By ew TTS eR oy GH Perso ea ‘m'= 1g, seater wa Teh 8 | ae we HH A eine! chs Pear enor wat aah B | GH eer Air eee 8 | ga we a) ate A SIR “USt mis' aH dae wis! aa 8) ale ael yoru ae g’B she RS staray HE faa twa ah 81 os are om rg we ogy A fret oe “TO see att x ara ae 40 J=106m, ope jo parallel vertical metallic rails AB and CD are separated by 1 m. They are connected at the two “nds by resistance R, and R, as shown in the figure. A horizontal metallic bar L of mass.0.2 kg slides without friction, vertically down the rails under the action of gravity. There is a uniform horizontal magnetic field of 0.6T perpendicular to the plane of the rails. It is observed that when the terminal velocity is attained, the power dissipated in R, and R, are 0.76 W and 1.2 W respectively. If the terminal velocity of bar Lis x m/s and R, is y © and R,is ZO then find the value of x + 76y + 10z. (g = 9.8 m/s?) (VEE -94] 2) aA weal aTgoll TERR AB clay CD Ger AR a tm Fh GA oR BI arg Gre ata AA aE TR, oa R, 8 GS 81 O.2kg aA eho ws L, Hee o wera H eRe ow eae AN A lve Fr ein Pero 8) 81 ei O.6T ar arrHT yaaa Aa HERB ae B raat Rear 8 1 ora aE MAT A HT EAN ah R, ser RH ate are wR: 0.76 W THT 1.2W 81 aR Os Lan AAT TT xm/s TH RTA OTR, BA ZO ah x + 7By + 102 1 A TET AAFTE | (g = 9.8 mis?) Appa gc Emp dun to A Rotating Rock :- yo x x x B dé= bl)or & x“ Be € sf = [award 4 ° ee x E= auofar]" £- 1bwh 2 &. rod of length 10 cm made up of conducting and non-conducting material (shaded partis non-conducting). The rod is rotated with constant angular velocity 10 rad/sec about point O, in constant and uniform magnetic field of 2 Tesla as shown in the figure. The induced emf between the pointA and B of rod will be 1Oom ah ws , wee cen sree vere S aang one 8 (wraifa arr sare &) | os Ofte & ofa: Pea wri a1 10 rad/sec'S vere Pract gadia da 22aa1 # Prague wh ae eh 8) vs SATB ital 3 aa ‘faerarcre EFT | V2 Iitanal naistamce of rad s a @8 bs OL Find Curmmnt ia K. ef _/ Ea > KR é a £ 5 D>‘ KER ae c= Lbol® a bud” Exlinaion '~ kek i raistome hos Bult Led D |! 2R 7 2 feds 3 i= Bet ——| 2A a é aj BwR inne igure there are wo Kenta conducting rods acho length’ rotating wih angular speed in ‘the directions shown. One end of each rod touches a conducting ring. Magnetic field B exists perpendicular to the plane of the rings. The rods, the conducting rings and the lead wires are resistanceless. Find the magnitude and direction of current in the resistance R. fea 8 2) yo WAM maw ve, yew A MUL ‘a’ SoA aa oS eeiA ag Re A YR eA ei velw oS 21 yw RR are aera wh BMT | yah da B cel oe rea Rema & | we, mew ae vie sitios ory aftedis @ | afte RA are ar var a Ren gra 7a | £=1BwAa we OV avs L&2 ? ov= 48vR find DV bbwom donut & bi a E=2BvUR Q Bon = ArPETIT Liss smooth conducting loop of radius ¢ = 1.0 m & fixed in a horizontal plane. A conducting rod of mass. mn LOKgandiongivstghy greater han ringed atthe cer ofthe oop can eat nthe zn plane such that the free end slides on the rim of the loop. There is a uniform magnetic field of strength B= 1.0 Téreced vertealy downers, Torodiertaled withanguarveliy n= .0rads anda. The feed end ofthe rod andthe im of hoop are comected Waugh abate of em. E,a fessor of resistance R = 1.0 Q, and initially uncharged capacitor of capacitance C= 1.0 F in series. Find Liga ara 99 8 foraat fiver = 41.098. cen ag afta § Rent #1 WH m= 1.0kg RAAT Te ES EU TS amram os ot go eB frets & Ato ge A ge oe Woe 8 fh gee Aa fare wre GT a RT arent oem 8) ae Ve wa Gradia aa ferwet diac = 1.072, wate te a awe ora 819s ©, =1.0rads% area iS Earth ort & an fix ots ord @ vs & fter fg cen qa #1 fer @ E, ara arefl 4e8 wa R= 1.00, & sfete 8 sige 8 cer ARE A srrafere eenkta fore aa C= 1.0F 8 a1 Sha ‘A oiga 8) varga () the time dependence of e.m4. E such that the current Ip = 4.0 Ain the circuit is constant. rane wt wr oy Pricer af ofteer Herat = 1.0 AF ERT Fe | (i) energy supplied by the battery by the time rod stops Bs S wat ve S ura F deg are ah wh wat 1B (w,- xt) Lt £-iR- it = ; c ol gl (», - 218e) rE -iR-it -9 Lt 2 c F=iktit — a *(w,- Bist ) Cc Es + t yy fotthng Vobuns

[ee at 0 W=h- «xt > O= WE D>t= % > t= kmw, of 318 futing Vals , we et W/ = T als . AW: = § wire loop enclosing a semi-circle of radius sm is located on the boundary of a uniform magnetic field of induction B=1T (Figure). At the moment: ‘the loop is set into rotation with a constant angular acceleration [i = 2 rad/sec’ about an axis O coinciding with a line of vector B on the boundary. The emf induced in the loop as a function of time tis [x x 10“ (—1)" * t] V, where n = 1, 2, .... is the number of the haif-revolution that the loop performs at the given moment t. Find the value of x. (The arrow in the figure shows the emt direction taken to be positive, att = 0 loop was completely outside) Prem a= Zom i sg ae BU YH AIA Gea Aa BHATT a a we Reals & (Ria) | t= O aver we qa Fran aioita caer B= Zradisec? a ae w afta tar BS war oi OF ae: GT YR Tez a7 ARa Roaous aa td wor B wa A [xx 104 Ay xYVE, oe n= 4,2,.... 8 a Re ta ae ga Soe gol A em wT galls wea Bx BATT aI BAT) (Pa A Ae A Ra, I ftomoms a errors fen wt gufer 81 (t= O—T aT ATER em) | = 2a 2 = Lxtxatx 4xiot a € = 4t xlot] DS x=4 Bon- APPETIT Qa ‘metal rod of mass m can rotate about a horizontal axis O, sliding along a circular conductor of radius (Fig). The arrangement is located in a uniform magnetic field of induction B directed perpendicular to the ring plane. The axis and the ring are connected to an emf source to form a circuit of resistance R. ‘Neglecting the friction, circuit inductance, and ring resistance, find the law according to which the source emfmust vary to make the rod rotate with a constant angular velocity Tag 2 oo “4 ™“t bat (E+ Lbwa') mg & tent AR “ E+ tbwar = gk den wl Ba C= ~ 1 boat Roinwt o d Ba ae ), A rod of length | is rotating with an angular speed « about its one end which is at a distance ‘a’ from an Z infinitely long wire carrying current . Find the emf induced in the rod at the instant shown in the figure. Qa rod of length | is rotating with an angular speed w about its one end on hich is at a distance ‘a’ from an infinitely long wire carrying current i. Find the emf induced in the rod at the instant shown in the figure. Brees d | a at, [-atan(es]] 2 Avs dg = Bund 1 dg. Mel wa ols arr (at nce) Lh &- Uiw ~ an wo 4 (at nus 0) = iw 8 fect anwss (atrws 9) “foop for THDVGHTS Falling Loop : ‘Arectangular loop of wire with mass m, width w, vertical length and resistance R falls out ofa magnetic field under the influence of gravity, as shown in Figure. The magnetic field is uniform and out ofthe paper Bi) within the area shown and zero outside of that area. At the time shown in the sketch, the loop is exiting the magnetic field at speed Le, A (a) Whatistheditection of the curent flowing in the circuit at the time shown, clockwise or counterclockwise? ‘Why did you pick this direction? (b) Using Faraday’s law, find an expression for the magnitude of the emf in this circuit in terms of the {quantities given. What is the magnitude of the current flowing in the circuit atthe time shown? (©) Besides gravity, what other force acts on the loop in the { ditection? Give its magnitude and direction interms ofthe quantities given. (, (4) Assume thatthe loop has reached a“terminal velocity” and sno longer accelerating, Whatisthe magnitude of that terminal velocity in terms of given quantities? sq = CA (©) Show that at terminal velocity, the rate at which gravity f doing Work on the loop is equal to the rate at which energy is being dissipated in the loop through Joule heating. bz Lonalont Ama 2A No. af tin 2 i At $20 amghe biotin Ax 8 0. Fino EMF a function of time At omy tin t > Anat betwen 546 w= wt D- BA = 6A crwt Ez -Nd¢@ - -No BA tawt oe olé €2-NGA[-Ssinwt] dot ot €& = N6Aw Lin( t) L Lovesbt of AC dlyrarno @ Axis of 40k Loincida with G dinchon. Gostiction iu fl, roof Huns N. Loih teams through I80° obout its diamuda A ballistic. galvomowetin prearuines horge J flowing Hspagh it: Fino B Raiden a ®. 6. NOG > iRe Noo Be ot oy R- Noo +» 442 NOP oe wah > Y z df= 260A > Y= MBAs [B= YF a 2NA Generators : ‘One of the most important applications of Faraday’s law of induction is to generators and motors. A generator converts mechanical energy into electric energy, whilea motor converts electrical energy into ‘mechanical energy. ay y + Z f s ( *C4—\s je 7 4 Figure : (a) simple generator. (b) The rotating loop as seen from above. Figure (a) isa simple illustration ofa generator. It consists of an N-tum loop rotating in a magnetic field which is assumed to be uniform, The magnetic flux varies with time, thereby inducing an emf. From Figure (b), we sce thatthe magnetic flux through the loop may be written as %, BAcos0 = BAcosat “The ate of change of magnetic axis $0 - Baosinot ai Since thete are N tums in the loop, the total induced emi across the two ends ofthe loop is, so, NoGe TNBAdsin ot we connect the generator toa circuit which has. resistance R, then the current generated in the circuit [cl _ NBAo isgivenby RR sina ‘The current is an alternating current which oscillates in sign and has an amplitude |, =NBA@/R. The powerdelivered t this circuit is jeje NBA ge mot ‘On the other hand, the torque exerted on the loop is += UB sin 0=pB sin ox ‘Thus, the mechanical power supplied to rotate the loop is, P= 10= 4B asin ot ‘Since the dipole moment forthe N-turn current loop is. Pp NABo weNlA= = sinot Asexpected, the mechanical power putin is equal tothe electric power output, Induced Electric Field : We have seen that the electric potential difference between two points 4 and B in an electric field can be written as AV=V,-V,=-[E 8 When the electric field is conservative, asis the case of electrostatics, the line integral of is path-independent, which implies FE . d8 =0 Faraday’s law shows that as magnetic flux changes with time, an induced current begins to flow. What causes the charges to move? Its the induced emf which is the work done per unit charge. However, since magnetic field can do no work, as we have shown, the work done on the mobile charges must be electric, and the electric field in this situation cannot be conservative because the line integral of a conservative field must vanish, Therefore, we conclude that there is a non-conservative electric field E,, associated with an induced emf: e. ‘Combining with Faraday’ law then yields PEs The above expression implies thata changing magnetic flux will induce anon-conservative electric field which can vary with time Iis important to distinguish between the induced, non-conservative electric field and the conservative electric field which arises from electric charges. ‘Asan example, let's consider a uniform magnetic field which points ito the page and is confined toa circular region with radius R, as shown in Figure. Suppose the magnitude of jj increases with time, ie., dB/dt> 0. Let’s find the induced electric field everywhere due to the changing magnetic field. Since the magnetic felis confined toacrcular region, from symmetryargumenis we choose the integration path to be acircle of radius r. The magnitude of the induced field E,. atall points ona circle is the same. According to Lenz’s law, the direction E,. of must be such that it would drive the induced current to produce a magnetic ficld opposing the change in magnetic flux. With the arca vector {pointing out of the page, the magnetic flux is negative or inward. With dB / dt> 0, the inward magnetic flux is increasing, Therefore, to counteract this change the induced current must flow counterclockwise to produce more outward flux. The direction of E,. is shown in Figure. ‘gure : Induced clectrc field due to changing magnetic flux Let’s proceed to find the magnitude of i, . Inthe region r_ dé Pod = ~ 2% $é = Lea: - ARB f a E-an = ~7R Ab dt a -R db an at ans O ix a hugh potuitio — ate ret) Qe” x kx dvc -Fuaodx ove JE csodx to = . Fen db a Vv athy B 2 dt aw £92 4% J AV= |r Bb, wOdu tp doz de Ja qsuro A L a = [RP soe gona m0 7 a vo - L “ a- yd = Boy [sure ao = 4 2 Kk we Le - &}b ton & = by (tme} = Ped teat [tm | = Bo yX & L ae ors z q We RC Circuit in a Magnetic Field ; Considera circular loop of wire oTradius Tying in they plane, as shown in Figure, The loop contains a resistor Rand a capacitor C, and is placed in a uniform magnetic field which points into the page and decreases at a rate dB/dt , with o> 0. Figure : RC circuit ina magnetic field (a) Find the maximum amount of charge on the capacitor. (b) Which plate, « orb, has higher potential? What causes charges to separat. a * foop for TrovGuTs EMF Due toa Time-Varying Magnetic Field : (a) () © @ A.uniform magnetic field jis perpendicular to a one-tum circular loop of wire of negligible resistance, asshownin Figure. The field changes with time as shown (the z direction is outof the page). The loop is ofradius r= 50 cm and is connected in series with a resistor of resistance R= 20 0. The around the circuit is indicated in the figure. What is the expression for EMF in this circuit in terms of B,(t) for this arrangement? Plotthe EMFin the circuit asa function of time, Label the axes quantitatively (numbers and units), Watch the signs. Note that we have labeled the positive direction of the emf in the left sketch consistent with the assumption that positive j is out of the paper. [Partial Ans: values of EMF are 1.96 V, 0.0 V, 0.98 V] Plot the current through the resistor R. Label the axes quantitatively (numbers and units). Indicate with ‘arrows on the sketch the direction of the current through R during each time interval. [Partial Ans: values of current are 98 mA, 0.0 mA, 49 mA] Plot the rate of thermal energy production in the resistor. [Partial Ans: values are 192 mW, 0.0 mW, 48 mW] do SEE From meRT Eddy Currents Suppose we replace the conducting loop of Fig. 30-8 with a solid conducting pine plate-If we then move the plate out ofthe magnetic field as we id the loop op (Fig. 30-102). the relative motion of the field and the conductor again induces a current in the conductor. Thus, we again encounter an opposing force andl must Cs do work because of the induced current. With the plate, however, the conduc- : tion electrons making up the induced current do not follow one path as they do with the loop. Instead, the eleetrons swiel about within the plate as if they ‘were caught in an eddy (whirlpool) of water. Such a current is called an eddy current and can be represented, as it isin Fig. 30-10a, as if if followed a single path, ‘AS with the conducting loop of Fig. 30-8, the current induced in the plate results in mechanical energy being dissipated as thermal energy. The dissipa- tion is more apparent in the arrangement of Fig. 30-10b; a conducting plate, free to rotate about a pivot, is allowed to swing down through a magnetic field like a pendulum. Each time the plate enters and leaves the field, a portion of its mechanical energy is transferred to its thermal energy. After several swings, no mechanical energy remains and the warmed-up plate just hangs fromits pivot. nee oteyeraieuserac [SiR ont ate ined in the plate. Atypical "The figure shows four wire loops, with ede lengths of ether Lor 2L. All fur loops aston epee Tin ms tprare ee eee impatience isco | Se a eaten case es Scertieialetorentcnge (imme eael een Reticle sea SELF InpucTANCEe SelfInductance : Consider again a coil consisting of V turns and carrying current /in the counterclockwise direction, as shown in Figure. Ifthe current is steady, then the magnetic flux through the loop will remain constant. However, suppose the current / changes with time, then according to Faraday’s law, an induced emf will arise to oppose the change. The induced current will flow clockwise ifdl/dt> 0, and counterclockwise if dl /dt<0, The property of the loop in which its own magnetic field opposes any change in current is called “self-inductance,” and the emf generated is called the self-induced emf or back emf, which we denote as ¢,. All current-carrying loops exhibit this property. In particular, an inductor isa circuit element (symbol SH» ) which has a large self-induetance. xi, Inductance of a Solenoid : a Compute the self-inductance of a solenoid with tums, length f, and radius FR with a current / flowing through each turn, as shown in Figure. & sereinductance ofa Foroid : Calculate the self-inductance ofa toroid which consists of N turns and has a rectangular cross section, with inner radius a, outer radius b and height h, as shown in Figure (a), a ‘ie we F He r (@) df= Bdhusd = UNithh ath $= fds = MoMA fs de air 4 Bz UeNih yp 6 wT be (4) f= Ne > ma)» H(M ES) a,b >>(6-a) > fn (1+ b-8) = (= Le UN A (b-0) = UNA ara ara Ca many metres of a thin wire are required to manufacture a solenoid of length /, = 100 em and inductance L= 1.0 mH ifthe solenoid's cross-sectional diameter is considerably less than its length? L= Ue Nak x Totok Loaglh we = ark. Le UN TR oun L an = Ue Gr RN) Yoh fnew) = hh Ms an RN-= fiat Mo Total bangle of wide = loom “Oonl- APFETIT * 3.319. Find the inductance of a unit length of a cable consisting of two thin-walled coaxial metallic cylinders if the radius of the out- side cylinder is 1 = 3.6 times that of the inside one. The perme- ability of a medium between the cylinders is assumed to be equal to unity. ye 3.321. Calculate the inductance of a unit length of a double tape line (Fig. 3.92) if the tapes are separated by a distance h which is considerably less than their width b, namely, b/h = 50, 3.322. Find the inductance of a unit length of a double line if the az radius of each wire is n times less Ze than the distance between the axes of the wires. The field inside the wires is to be neglected, the per- meability is assumed to be equal to unity throughout, and 1 > 1. >. Bie 209 ¢ Indvc TOR». _amtim_ Uist > Hanes Enngy Saud in Tductor: (Magna. Full: od 1 Wont)” meh Ue veVIuMt by geen d Lea, = 4, +hth, Qe Is Ri 2 IN aE + nN a é = 4 IX Qe ~ v ' nN Nh v | RL Circuits Medkifizd Kerchopfs Loop Rule aa £-iR=0 é Kye, Pt ees 9 Wa Scan Lonvivkion '- 9 TP 2B einer — D 4V=VB-\h =~ Loe iH 6 at dicO were > OV= Vy -Vj a pa skig sunt ale : TD. a A > BV=\,-V, ah 6 fi 4 “tnaseng cunt . TD. ds Zo ee ie Be eooterr > av = Va" Vp = Ae At —k— dhsuing nn Growth. of Luant mn sui RE Lrewit - z XL At time t >> “4 ek €-ik - kde =5 > E-iR= fe ot ot é t = (ot 5 fn(E- balé=t6)] = E-iR x Sn (E-iRD-WE = ~ KE “L tm (E=th) = “et a iz ECr-e MH) = & (a) R j2@ , TH a dL A Tere corti . kee = hor = - od [-Ke Vi = oe =48 Co e ole Vv, _¢é ehh - te & Va stk = Eee) K Va = EC- ek) 2 (i-e 72) Ak tso 5 (20 Ys €, 420 C 4 RL -L” Copan Uincuit,) Ab tem i-£, Wr? , we € 77 (chou Hircuct,) UA Loncept '— Inclactor always spbnua o suddun “tos ity thang fan gl ne Pm Al ok we i, REL -t/t - t,e X circuit contains an ideal cell and an inductor with a switch. Initially the switch is open .Itis closed at t=0.Find the current as a function of time. . L — > a z “ln the following circuit, the switch is closed at t= 0. Find the currents i, iyi, and Satteo and att =<. Initially all currents are zero. Sane following circuit the switch is closed at = 0.Intially there is no current in inductor. Find out current the inductor coll as a function of time. _ i (r- & /*) fon time Lowatant : - v,, the circuit shown in the figure,, remains closed for a long time and S, remains open. Now S, is closed and S, is opened. Find out the di/dt just after that moment. - aR ce ~ -4€ Azer 0 8 Loe sige tug at Lod + E.ak+4E zo dt & oh dt L 7 Att= 0 switch is closed (shown in figure) after a long time suddenly the inductance of the inductor is made L ni fimes lesser (>) then ts inital value, find out instant current just after the operation Flum x tovanvid of tho vary mom - resistor conmectdin series Letthe sutch She osod att 0 Suppose a =0 current inthe inductor. then nd ot equation wre entasafuncton of tne a Meg fa Fo o €-ik- Al 29D E-iR= dt [ee eo) [i : fi ; te Gi) -d CE-LO ae Rte ls 6-(€-GR€ R nthe circuit diagram shown in the figure the switches S, and S, are closed at time t = 0. After time t= (0.1) én 2sec, switch S, is opened. The current in the circuit at time , t = (0.2) In 2 sec is equal x to 35 amp. Findout value of x. Pra Rane Roe erat = Owe aftr SIS, a% 81 I t= (0.1) (n2seo, Ta gH S, w Gea 1 uftoes ora t= (0.2)In2seo. we ATF amp eat x wT AT ATA wR . lon itt Ge) looy = 102 = oA iH to Trt ue. 10 — lot , b= foCi-e ) = to Cy- en (re itnt y 2 Le Ch =loti-e**) = loli-e*™*™) t= lo (1-4) > |is SA 2 ty for tou-F Come - S, u opunsd at t= ola Sos Loo ° 1h >is gE uae RX i= Joo-(loo -asy ,- So i dost ie SPR Oe 2 be ails) ul BR + wo na 7A 2 Qin the gure, @ conducting od oftength ¢= 1 meter and mass m = 1 kp moves wih inital velocity 4u=5mmis. ona fixed horizontal frame containing inductor L = 2 H and resistance R = 12. PQ and MN are. smooth, conducting wires. There is a uniform magnetic field of strength B = 17, Initially there is no current in the inductor. Find the total charge in coulomb, flown through the inductor by the time velocity of rod becomes v,= 1 m/s and the rod has travelled a distance x= 3 meter. PRargUR (= 1 meters, m=1 kg xaMTA Go VIET US ARP WT U=S mise ww AIT Re she Fe afd oe <8 | gH SoH dS TA R= 1221 GH MPA She L= ZHI GH Heeped oN ReaTGUR GSI oT BL Paste MN fea aera ae & | RATT H ces GTA Graeha AB = AT shan gar wee A Rew Que Bah carer Ei a @8h ch oa EK = 3 meter ge eh A sie Sera a v,= A mis a are, Ta eT Hew eT A fer oer (agi) HR eH et 7 M N as EM i A Hay Oly = Lda re ae uv ly [Ab= -mdu t = L fae, B foat Jat iz ut ke Bix = hu =Ld emdy 2 Bh + Ble bhx = ve brat & ‘ R t 4 _ afte . bi fndk + BAv at = BL [ude Ba Lh ho a” -m(v-u)= V + eh x BL & qe mG) - Ble BL R hacy ovtah QA metal rod O4 of mass mand rays kept rotating witf/a constant angular speed « in a 1 plane about a horizontal axis at the end Q as shownin figure Theftee end A is arranged to slide without friction along a fixed conducting circular ring in the same planeas that of rotation, A uniform and constant magnetic induction B is applied perpendicular and into the plane of rotation as shown in figure. ‘An inductor and un externl resistance R are connected through a switch S between the point O and a point C on the ring to form an electrical circuit, Neglect the resistance ofthe ring and the rod, Initially, the switeh is open. (a) What is the induced EMF across the terminals of the switch? (b) The switch Sis closed at time ¢=0. Obtain an expression for the current as.a function of time. In the steady state, obtain the time dependence of the torque required to maintain the constant angular speed, given that the rod OA was along the positive X-axis at /=0. iS ? (@ Sar oe D Freq rem > Aws Te ito -L = Bwat (a ye RR ele pani o

You might also like